What is Canonical transformation: Definition and 55 Discussions

In Hamiltonian mechanics, a canonical transformation is a change of canonical coordinates (q, p, t) → (Q, P, t) that preserves the form of Hamilton's equations. This is sometimes known as form invariance. It need not preserve the form of the Hamiltonian itself. Canonical transformations are useful in their own right, and also form the basis for the Hamilton–Jacobi equations (a useful method for calculating conserved quantities) and Liouville's theorem (itself the basis for classical statistical mechanics).
Since Lagrangian mechanics is based on generalized coordinates, transformations of the coordinates q → Q do not affect the form of Lagrange's equations and, hence, do not affect the form of Hamilton's equations if we simultaneously change the momentum by a Legendre transformation into





P

i


=




L







Q
˙




i





.


{\displaystyle P_{i}={\frac {\partial L}{\partial {\dot {Q}}_{i}}}.}
Therefore, coordinate transformations (also called point transformations) are a type of canonical transformation. However, the class of canonical transformations is much broader, since the old generalized coordinates, momenta and even time may be combined to form the new generalized coordinates and momenta. Canonical transformations that do not include the time explicitly are called restricted canonical transformations (many textbooks consider only this type).
For clarity, we restrict the presentation here to calculus and classical mechanics. Readers familiar with more advanced mathematics such as cotangent bundles, exterior derivatives and symplectic manifolds should read the related symplectomorphism article. (Canonical transformations are a special case of a symplectomorphism.) However, a brief introduction to the modern mathematical description is included at the end of this article.

View More On Wikipedia.org
  1. A

    I Does the Hamilton-Jacobi equation exist for chaotic systems?

    Given a Hamiltonian ##H(\mathbf{q},\mathbf{p})##, in the time-independent Hamilton-Jacobi approach we look for a canonical transformation ##(\mathbf{q},\mathbf{p})\rightarrow(\mathbf{Q},\mathbf{P})## such that the new Hamiltonian is one of the new momenta...
  2. Lagrange fanboy

    I Proof that canonical transformation implies symplectic condition

    Goldstein's Classical Mechanics makes the claim (pages 382 to 383) that given coordinates ##q,p##, Hamiltonian ##H##, and new coordinates ##Q(q,p),P(q,p)##, there exists a transformed Hamiltonian ##K## such that ##\dot Q_i = \frac{\partial K}{ \partial P_i}## and ##\dot P_i = -\frac{\partial...
  3. LCSphysicist

    Time dependent canonical transformation

    THe question is pretty simple. I was doing an exercise, in which $$p = \lambda P, Q = \lambda q$$ is a canonical transformation. We can check it by $$\{Q,P \} = 1$$ But, if we add $$t' = \lambda ^2 t$$, the question says that the transformation is not canonical anymore. I am a little...
  4. L

    I Canonical transformation vs symplectomorphism

    I have read that canonical transformation is basically a symplectomorphism which leaves the symplectic form invariant. My understanding is that the canonical transformation is a passive picture where we keep the point on the phase space fixed and change the coordinate chart, where...
  5. Saptarshi Sarkar

    Change in Hamiltonian under Non-Canonical transformation

    I know that if the transformation was canonical, the form of Hamilton's equation would remain invariant. If the generating function for the transformation was time independent, then the Hamiltonian would be invariant and we could directly replace q and p with the transformation equations to get...
  6. D

    Phase space of a harmonic oscillator and a pendulum

    Hello everybody, new here. Sorry in advance if I didn't follow a specific guideline to ask this. Anyways, I've got as a homework assignment two cannonical transformations (q,p)-->(Q,P). I have to obtain the hamiltonian of a harmonic oscillator, and then the new coordinates and the hamiltonian...
  7. dRic2

    Canonical transformation in classical mechanics

    I'm stuck from the beginning. I though I understood the difference between ## \delta## and ##d##, but apparently I was wrong, because I don't know how to exploit it here... Any hint would be greatly appreciated Thank Ric
  8. gasar8

    Canonical invariance vs. Lorentz invariance

    Homework Statement I have an assignment to prove that specific intensity over frequency cubed is Lorentz invariant. One of the main tasks there is to prove the invariance of phase space d^3q \ d^3p and I am trying to prove it with symplectic geometry. I am following Jorge V. Jose and Eugene J...
  9. A

    How can I improve the legibility of my homework post for better responses?

    Homework Statement [/B]Homework EquationsThe Attempt at a Solution [/B] From Poisson bracket relation I have arrived at this point Can anyone please suggest to proceed further
  10. Vicol

    A Canonical transformation - derviation problem

    Let me show you part of a book "Mechanics From Newton’s Laws to Deterministic Chaos" by Florian Scheck. I do not understand why these integrands can differ by more than time derivative of some function M. Why doesn't it change the value of integrals? It seems this point is crucial for me to...
  11. A

    How can I use Poisson bracket to find P in a canonical transformation?

    Homework Statement q,p transforms canonicaly to Q,P where given Q=q(t+s)+(t+s)p ,t is time and s is constt To find P Homework Equations Poisson bracket {Q,P}qp=1 The Attempt at a Solution Using Poisson bracket I find (t+s)*(dP/dp-dP/dq)=1
  12. thecourtholio

    Hamiltonian conjugate dynamic variables

    Homework Statement Consider a charge ##q##, with mass ##m##, moving in the ##x-y## plane under the influence of a uniform magnetic field ##\vec{B}=B\hat{z}##. Show that the Hamiltonian $$ H = \frac{(\vec{p}-q\vec{A})^2}{2m}$$ with $$\vec{A} = \frac{1}{2}(\vec{B}\times\vec{r})$$ reduces to $$...
  13. B

    A Test if 2nd order diff eq. can be derived from a Hamiltonian

    Imagine I have a complicated second-order differential equation that I strongly suspect can be derived from a Hamiltonian (with additional momentum dependence beyond p2/2m, so the momentum is not simply mv, but I don't know what it is). Are there any ways to test whether or not the given...
  14. CassiopeiaA

    A Symplectic Condition For Canonical Transformation

    I am reading Chapter 9 of Classical Mech by Goldstein.The symplectic condition for a transformation to be canonical is given as MJM' = J, where M' is transpose of M. I understood the derivation given in the book. But my question is : isn't this condition true for any matrix M? That is it doesn't...
  15. F

    I Canonical transformations and generating functions

    I've been reading about canonical transformations in Hamiltonian mechanics and I'm a bit confused about the following: The author considers a canonical transformation $$q\quad\rightarrow\quad Q\quad ,\quad p\quad\rightarrow\quad P$$ generated by some function ##G##. He then considers the case...
  16. F

    Hamiltonian as the generator of time translations

    In literature I have read it is said that the Hamiltonian ##H## is the generator of time translations. Why is this the case? Where does this statement derive from? Does it follow from the observation that, for a given function ##F(q,p)##, $$\frac{dF}{dt}=\lbrace F,H\rbrace +\frac{\partial...
  17. S

    The restricted canonical transformation group

    Homework Statement Show that the set of restricted canonical transformation forms a group. Verify this statement once using the invariance of Hamilton's principle under canonical transformation, and again using the symplectic condition. Homework Equations (Invariance of Hamilton's principle...
  18. kolawoletech

    A Most General form of Canonical Transformation

    How do I go about finding the most general form of the canonical transformation of the form Q = f(q) + g(p) P = c[f(q) + h(p)] where f,g and h are differential functions and c is a constant not equal to zero. Where (Q,P) and (q,p) represent the generalised cordinates and conjugate momentum in...
  19. S

    Canonical Transformation (two degrees of freedom)

    Homework Statement Point transformation in a system with 2 degrees of freedom is: $$Q_1=q_1^2\\Q_2=q_q+q_2$$ a) find the most general $P_1$ and $P_2$ such that overall transformation is canonical b) Show that for some $P_1$ and $P_2$ the hamiltonain...
  20. S

    Finding canonical transformation

    Homework Statement If in a system with i degrees of freedom the $$Q_i$$ are given what is the best way to proceed for finding the $$P_i$$ so that we have an overall canonical transformation. say for a two degree freedom system we have $$Q_1=q_1^2 $$ and $$ Q_2=q_1+q_2$$ Homework Equations...
  21. R

    Fundamental Poisson Bracket - Canonical Transformation

    When proofing the poisson brackets algebraically, what is the tool of choice. Can one use the muti dimensionale chain rule or how is it usally done?
  22. D

    Finding the generator of a transformation

    Homework Statement Consider ##\mathscr{H} = \frac12 p^2 + \frac12 x^2, ## which is invariant under infinitesimal rotations in phase space ( the ##x-p## plane). Find the generator of this transformation (after verifying that it is canonical). Homework EquationsThe Attempt at a Solution So the...
  23. D

    Showing infinitesimal transformation is canonical

    Homework Statement So we have infinitesimal transformations from ##q_i## to ##\bar{q_i}## and ##p_i## to ##\bar{p_i}## ( where ##p_i## represents the canonical momentum conjugate of ##q_i##) given by $$\bar{q_i} = q_i + \epsilon \frac{\partial g}{\partial p_i}$$ $$\bar{p_i} = p_i - \epsilon...
  24. P

    Non-canonical form into canonical transformation 1-d partial dif.

    Homework Statement Problem 29. Use the subtraction trick U(tilda) = U−U1 to reduce the following problems with non-canonical boundary conditions to the canonical ones and write down the equations in terms of the variable ˜u (do not solve them). Note that there are infinitely many u1’s that...
  25. Z

    Canonical Transformation / Poisson Brackets

    Question: (A) Show that the following transformation is a canonical transformation: Q = p + aq P = (p - aq)/(2a) (B) Find a generating functions for this transformation. Attempt of Solution: Alright, so this seems to be a very straight forward problem. Transformations are canonical...
  26. M

    Given a canonical transformation, how does one find its type?

    I'm given the following transformation X=x \cos \alpha - \frac{p_y}{\beta} \sin \alpha Y=y \cos \alpha - \frac{p_x}{\beta} \sin \alpha P_X=\beta y \sin \alpha + p_x \cos \alpha P_Y=\beta x \sin \alpha + p_y \cos \alpha and I'm asked to find what type(s) of transformation it is. I'm not...
  27. M

    Canonical transformation problem

    Homework Statement Let Q^1 = (q^1)^2, Q^2 = q^1+q^2, P_{\alpha} = P_{\alpha}\left(q,p \right), \alpha = 1,2 be a CT in two freedoms. (a) Complete the transformation by finding the most general expression for the P_{\alpha}. (b) Find a particular choice for the P_{\alpha} that will reduce the...
  28. M

    Question about canonical transformation

    I was going through my professor's notes about Canonical transformations. He states that a canonical transformation from (q, p) to (Q, P) is one that if which the original coordinates obey Hamilton's canonical equations than so do the transformed coordinates, albeit for a different Hamiltonian...
  29. darida

    Verifying a Canonical Transformation with Poisson Brackets

    Homework Statement Show that Q_{1}=\frac{1}{\sqrt{2}}(q_{1}+\frac{p_{2}}{mω}) Q_{2}=\frac{1}{\sqrt{2}}(q_{1}-\frac{p_{2}}{mω}) P_{1}=\frac{1}{\sqrt{2}}(p_{1}-mωq_{2}) P_{2}=\frac{1}{\sqrt{2}}(p_{1}+mωq_{2}) (where mω is a constant) is a canonical transformation by Poisson bracket test. This...
  30. L

    Infinitesimal Canonical Transformation

    Hi all! Another questions which is due to the gaps in my calculus knowledge. In these notes: http://people.hofstra.edu/Gregory_C_Levine/qft.pdf in the line above eq. (1) where it says that notation P is now unecessary, is it because \partial{ (p+\delta p)} is much smaller than p+\delta p...
  31. A

    Understanding the Concept of Canonical Transformation in Hamiltonian Mechanics

    There's a part in my book that I don't understand. I have attached the part and it is basically about how to transform from a set of conjugate variables (q,p) to another (Q,P) while preserving the hamilton equations of motion. I don't understand what he means by q,Q being separately independent...
  32. A

    Canonical transformation for Harmonic oscillator

    Find under what conditions the transformation from (x,p) to (Q,P) is canonical when the transformation equations are: Q = ap/x , P=bx2 And apply the transformation to the harmonic oscillator. I did the first part and found a = -1/2b I am unsure about the next part tho: We have the...
  33. A

    Show condition for canonical transformation

    Homework Statement Consider the transformation from the variables (q,p) to (Q,P) by virtue of q = q(Q,P), p = p(Q,P) and H(q,p,t) = H(Q,P,t). Show that the equations of motion for Q,P are: \partialH/\partialQ = -JDdP/dt \partialH/\partialP = JDdQ/dt where JD is the Jacobian determinant...
  34. M

    Exploring Canonical Transformations

    I have posted before this, an example in which I struggled through. Now am gnna ask something more general, for me and for the students who suffer from studying a material alone. If you were asked to prove that the time-independent transformation P=.. and Q=.. is canonical. And finding the...
  35. fluidistic

    Hamiltonian, generating function, canonical transformation

    Homework Statement Consider a harmonic oscillator with generalized coordinates q and p with a frequency omega and mass m. Let the transformation (p,q) -> (Q,P) be such that F_2(q,P,t)=\frac{qP}{\cos \theta }-\frac{m\omega }{2}(q^2+P^2)\tan \theta. 1)Find K(Q,P) where \theta is a function of...
  36. fluidistic

    Finding a generating function for a canonical transformation

    Homework Statement I'm trying to find a generating function for the canonical transformation Q=\left ( \frac{\sin p}{q} \right ), P=q \cot p.Homework Equations I am not really sure. I know there are 4 different types of generating function. I guess it's totally up to me to choose the type of...
  37. C

    Canonical transformation between two given hamiltonians

    Hello everyone, I am given the inital hamiltonian H = (1/2)*(px2x4 - 2iypy + 1/x2) and the transformed hamiltonian K = (1/2)*(Px2 + Py2 + X2 + Y2) and I'm supposed to show there exists a canonical transformation that transforms H to K and find it. I don't know how to solve problems of this sort...
  38. maverick280857

    Canonical Transformation of the Hubbard Model

    Hi, Suppose we have a 2 site Hubbard model, with the hopping Hamiltonian given by H_t and the Coulomb interaction Hamiltonian given by \hat{H}_U. In the strong coupling limit (U/t >> 1), we define a canonical transformation of \hat{H} = \hat{H}_U + \hat{H}_t, as H' =...
  39. J

    Canonical transformation in Hamiltonian

    Hamiltonian H=\frac{1}{2m}(P+\frac{e}{c}A)^{2} - e\phi and H^{'}=\frac{1}{2m}(P+\frac{e}{c}A^{'})^{2} - e\phi^{'} With gauge: A^{'}=A+\nabla\chi and \phi^{'}=\phi-\frac{1}{c}\dot{\chi} Why H^{'}-\frac{e}{c}\dot{\chi}=e^{-\frac{ie\chi}{\hbar c}}He^{\frac{ie\chi}{\hbar c}} ? Thanks.
  40. I

    BCS theory by canonical transformation

    I am reading Tinkham's "introduction to superconductivity" 1975 by McGraw-Hill, Inc. Tinkham derives the BCS theory by canonical transformation. At the beginning of the chapter he writes: "We start with the observation that the characteristic BCS pair interaction Hamiltonian will lead to a...
  41. S

    Canonical Transformation and harmonic-oscillator

    Show that the transformation Q = p + iaq , P = (p-iaq)/2ia is canonical and find the generating function. Use the transformation to solve the harmonic-oscillator problem. I was able to determine if the transformation is canonical, and it is. However, when it came to finding the...
  42. L

    Generating function for canonical transformation

    Homework Statement Given the transformation Q = p+iaq, P = \frac{p-iaq}{2ia} Homework Equations find the generating function The Attempt at a Solution As far as I know, one needs to find two independent variables and try to solve. I couldn't find such to variables. I've...
  43. R

    Is a Rotated Frame a Canonical Transformation in Classical Mechanics?

    Homework Statement Verify that the change to a rotated frame is a canonical transformation: \bar{x} = x cos\theta - y sin\theta \bar{y} = x sin \theta + y cos \theta \bar{p_x} = p_x cos \theta - p_y sin\theta \bar{p_y} = p_x sin \theta + p_y cos \theta Where [f,g] = poisson bracket Homework...
  44. S

    How Can I Verify a Canonical Transformation Using the Poisson Bracket?

    Homework Statement Verify that q_bar=ln(q^-1*sin(p)) p_bar=q*cot(p) * represents muliplication sorry i don't know how to use the programming to make it look better 2. The attempt at a solution my problem is that i really have no clue what is going on. I have read...
  45. N

    Canonical Transformation Problem

    Homework Statement Show that the time reversal transformation given by Q = q, P = − p and T = − t, is canonical, in the sense that the form of the Hamiltonian equations of motion is preserved. However, it does not satisfy the invariance of the fundamental Poisson Bracket relations. This is...
  46. S

    Calculating generator function in canonical transformation

    I'm searching for an example of how to find out generator function for a canonical transformation, when new canonical variables are given in terms of old variables. Any help is greatly appreciated.
  47. strangerep

    Most General Canonical Transformation?

    In classical Hamiltonian mechanics, the concept of a canonical transformation ("CT") preserving the form of Hamilton's eqns is well known. Textbooks (e.g., Goldstein) distinguish "restricted" CTs that just mix the q's and p's (generalized coordinates and generalized momenta respectively)...
  48. maverick280857

    Canonical Transformation of Parabolic PDEs

    Hi again I am studying PDEs and came across a solved problem in my textbook, which describes the transformation of a parabolic second order PDE to canonical form. I want to know how to find the second canonical substitution when one has been computed from the characteristic equation...
  49. M

    Extended Canonical Transformation

    Homework Statement The transformation equations are: Q=q^\alpha cos(\beta p) P=q^\alpha sin(\beta p) For what values of \alpha and \beta do these equations represent an extended canonical transformation? The Attempt at a Solution Well, just for a start, what is the condition for a...
  50. L

    How Does a Canonical Transformation Relate to Hamilton's Equations of Motion?

    Homework Statement Consider a canonical transformation with generating function F_2 (q,P) = qP + \epsilon G_2 (q,P), where \epsilon is a small parameter. Write down the explicit form of the transformation. Neglecting terms of order \epsilon^2 and higher,find a relation between this...
Back
Top